This set of exercises is retrieved from the eighth chapter of Linear Algebra by Robert J. Valenza. Note that these solutions are not fully elaborated; You have to fill the descriptions by yourself.
Problem 8.1
Using the recursive definition given in the proof of the existence of determinant, systematically evaluate the determinant of the following matrix:
\[A=\begin{pmatrix}1&2&1\\0&1&1\\1&0&2\end{pmatrix}.\]
Solution. \[\begin{aligned} \det (A) &= 1 \cdot \det \left[\begin{array}{cc} 1 & 1 \\ 0 & 2 \end{array}\right] - 2 \cdot \det \left[\begin{array}{cc} 0 & 1 \\ 1 & 2 \end{array}\right] + 1 \cdot \det \left[\begin{array}{cc} 0 & 1 \\ 1 & 0 \end{array}\right] \\[6pt] &= 1 \times 2 - 2 \times (-1) + 1 \times (-1) = 3. \end{aligned}\]
Problem 8.2
For any angle \(\theta,\) evaluate the determinant of the matrix
\[M_\theta = \left(
\begin{array}{cr} \cos\theta & -\sin \theta \\ \sin \theta & \cos \theta \end{array}
\right).\]
Recall that this is the matrix of rotation about the origin by the angle \(\theta.\)
Solution. \[\det ( M_\theta ) = \cos^2 \theta -(-\sin ^2 \theta ) = 1 .\]
Problem 8.3
Show that for \(A\in M_n (K)\) and \(\lambda \in K,\) \(\det (\lambda A) = \lambda^n \det(A).\)
Solution. Let \(A^j\) be the \(j\)th column of \(A,\) that is, \[A = (A^1 ,\, A^2 ,\, A^3 ,\, \cdots ,\, A^n ).\] Then \[\begin{aligned} \det (\lambda A) &= \det((\lambda A^1 ,\, \lambda A^2 ,\, \lambda A^3 ,\, \cdots ,\, \lambda A^n )) \\[6pt] &= \lambda \det(( A^1 ,\, \lambda A^2 ,\, \lambda A^3 ,\, \cdots ,\, \lambda A^n )) \\[6pt] &= \lambda ^2 \det(( A^1 ,\, A^2 ,\, \lambda A^3 ,\, \cdots ,\, \lambda A^n )) \\[6pt] & \quad \quad \vdots \\[6pt] &= \lambda ^n \det(( A^1 ,\, A^2 ,\, A^3 ,\, \cdots ,\, A^n )) \\[6pt] &= \lambda ^n \det(A). \end{aligned}\]
Problem 8.4
Show that if a matrix has a row or column of \(0 \text{'s,}\) then its determinant is \(0.\)
Solution. Let \(A = (a_{ij})\in M_n (K)\) and suppose that \(p\)th row consists only of \(0,\) that is, \(a_{pj}=0\) for \(j=1,\,2,\,\cdots,\,n.\) Calculating the determinant of \(A\) along the \(p\)th row, we have \[\begin{aligned} \det(A) &= \sum_{j=1}^n (-1)^{p+j} a_{pj} \det(\partial_{pj} A) \\[4pt] &= \sum_{j=1}^n (-1)^{p+j} \cdot 0 \cdot \det(\partial_{pj} A) =0. \end{aligned}\]
Problem 8.5
Show that the determinant of a matrix is unchanged if we add a scalar multiple of one column to another.
Solution. Let \(A = (a_{ij})\in M_n (K),\) and let \(A^j\) be the \(j\)th column of \(A,\) that is, \[A = (A^1 ,\, A^2 ,\, \cdots ,\, A^n ).\] Consider the operation \[kA^p + A^q \,\rightarrow\, A_q \quad (p\ne q)\] on \(A\) and let \(B\) be the result. Define \[b_{ij} = \begin{cases} k & \text{if} \, (i,\,j) = (p,\,q) \\[5pt] 0 & \text{otherwise,} \end{cases}\] and take \[R = (r_{ij})_{n\times n}= I_n + (b_{ij})_{n\times n}.\] Then we have \[AR = B.\] But for \[\det(\partial_{pq} R)=0\] and \[\begin{aligned} \det(R) &= \sum_{i=1}^n (-1)^{i+q} r_{iq} \det(\partial_{iq} R) \\[6pt] &= \sum_{i=1}^n (-1)^{i+q} \delta_{iq} \det(\partial_{iq} R) \\[6pt] &= \det(I_n) = 1, \end{aligned}\] we have \[\det(AR) = \det(A)\det(R) = 1\cdot \det(A) = \det(A).\] As an example, let \(n=3\) and consider \(A = (a_{ij})_{3\times 3}.\) Suppose that \(k\) is a nonzero real number and we are given the operation \[kA^2 + A^3 \,\rightarrow\, A^3 ,\] that is, the result of the operation is \[B = [A^1 ,\, A^2 ,\, kA^2 + A^3 ].\] Observe that \[AR=\left[\begin{array}{ccc} a_{11} & a_{12} & a_{13} \\ a_{21} & a_{22} & a_{23} \\ a_{31} & a_{32} & a_{33} \end{array}\right] \left[\begin{array}{ccc} 1 & 0 & 0 \\ 0 & 1 & k \\ 0 & 0 & 1 \end{array}\right] = [A^1 ,\, A^2 ,\, kA^2 + A^3 ].\] Since \[\begin{aligned} \det(R) &= \det\left[\begin{array}{ccc} 1 & 0 & 0 \\ 0 & 1 & k \\ 0 & 0 & 1 \end{array}\right] \\[6pt] &= 0 \cdot \det\left[\begin{array}{cc} 0 & 1 \\ 0 & 0 \end{array}\right] - k\cdot\det\left[\begin{array}{cc} 1 & 0 \\ 0 & 0 \end{array}\right] + 1\cdot\det\left[\begin{array}{cc} 1 & 0 \\ 0 & 1 \end{array}\right] \\[6pt] &= 0 + 0 + \det\left[\begin{array}{cc} 1 & 0 \\ 0 & 1 \end{array}\right] = 1 = \det(I_3 ), \end{aligned}\] we have \[\det(B) = \det(AR) = \det(A)\det(R) = \det(A)\cdot 1 = \det(A).\]
Problem 8.6
Consider the matrix
\[A = \begin{pmatrix}
x_1 & a_1 & b_1 \\
x_2 & a_2 & b_2 \\
x_3 & a_3 & b_3
\end{pmatrix}\]
in \(M_3 (K),\) where the \(a\text{'s}\) and \(b\text{'s}\) are fixed and the \(x\text{'s}\) may vary. Show that the set of all \((x_1,\,x_2,\,x_3)\) such that \(\det(A)=0\) is a subspace of \(K^3\) of dimension at least \(2.\)
Solution. Let \(A^j\) be the \(j\)th column of \(A.\)
If \(A^2,\) \(A^3\) are linearly dependent, then \(\det(A)=0\) for any \((x_1 ,\,x_2 ,\,x_3 ).\) Hence \[\left\{ (x_1 ,\,x_2 ,\,x_3 ) \,\vert\, \det(A) =0\right\} = \mathbb{R}^3.\] We now consider the case that \(A^1,\) \(A^2\) are linearly independent. Then \[\det(A)=0\] if and only if \[(x_1 ,\,x_2 ,\,x_3) = \lambda_2 A^2 + \lambda_3 A^3\] for some scalars \(\lambda_2\) and \(\lambda_3 .\) That is, \[\left\{ (x_1 ,\,x_2 ,\,x_3 )\,\vert\,\det(A)=0\right\} = \operatorname{Span}(A^2 ,\,A^3 ).\] Hence this set constitutes a \(2\)-dimensional subspace of \(\mathbb{R}^3.\)
Problem 8.7
Evaluate the determinant of the matrix
\[A=\begin{pmatrix}
0&1&0&0&0\\
0&0&1&0&0\\
0&0&0&0&1\\
1&0&0&0&0\\
0&0&0&1&0
\end{pmatrix}.\]
This is an example of a permutation matrix since it acts by permuting the canonical basis vectors of \(K^5.\)
Solution. \(A\) is expressible by \[A = [ {\mathbf{e}}_4 ,\, {\mathbf{e}}_1 ,\, {\mathbf{e}}_2 ,\, {\mathbf{e}}_5 ,\, {\mathbf{e}}_3 ].\] Since \[\left(\begin{array}{ccccc} 1 & 2 & 3 & 4 & 5 \\ 4 & 1 & 2 & 5 & 3 \end{array}\right)\] is an even permutation, we need even number of column-exchange operations to make \(A\) as the identity matrix. Hence we have \(\det(A) =1.\)
Problem 8.8
Evaluate the determinant of the following matrix using expansion by any row or column as appropriate.
\[A = \left(\begin{array}{ccrc}
1&1&-1&2 \\
0&1&2&0 \\
4&0&3&1 \\
0&2&0&0
\end{array} \right).
\]
Solution. Along the \(4\)th row, we have \[\begin{aligned} \det(A) &= -0 \cdot \det (\partial_{41} A) + 2 \cdot \det(\partial_{42} A) - 0 \cdot \det(\partial_{43} A) + 0\cdot \det(\partial_{44} A) \\[6pt] &= 2\cdot \det\left[\begin{array}{crc} 1 & -1 & 2 \\ 0 & 2 & 0 \\ 4 & 3 & 1 \end{array}\right] \\[6pt] &= 2 \times 2 \times \det\left[\begin{array}{cc} 1 & 2 \\ 4 & 1 \end{array}\right] \\[6pt] &= 2 \times 2 \times (-7) = -28. \end{aligned}\]
Problem 8.9
Evaluate the determinant of the following matrix using expansion by any row or column as appropriate.
\[A = \left(\begin{array}{ccrc}
0&1&-1&2 \\
1&1&2&0 \\
4&0&2&-1 \\
3&2&0&4
\end{array} \right).
\]
Solution. Along the \(2\)nd column, we have \[\begin{aligned} \det(A) &= -1 \cdot \det\left[\begin{array}{ccr} 1 & 2 & 0 \\ 4 & 2 & -1 \\ 3 & 0 & 4 \end{array}\right] +1 \cdot \det\left[\begin{array}{crr} 0 & -1 & 2 \\ 4 & 2 & -1 \\ 3 & 0 & 4 \end{array}\right] +2 \cdot \det\left[\begin{array}{crr} 0 & -1 & 2 \\ 1 & 2 & 0 \\ 4 & 2 & -1 \end{array}\right] \\[6pt] &= -1 \times (8-6-32) +1 \times (3+16-12) + 2\times (4-1-16) \\[6pt] &= -1 \times (-3) + 1\times 7 + 2\times (-13) = 11. \end{aligned}\]
Problem 8.10
Evaluate the determinant of the following matrix without expanding by either rows or columns.
\[A = \begin{pmatrix}
1&1&5 \\ 2&0&2 \\ 4&0&0
\end{pmatrix}.
\]
Solution. Using the rule of Sarrus, we have \[\begin{aligned} \det(A) &= 1 \times 0 \times 0 + 1 \times 2 \times 4 + 5 \times 2 \times 0 \\[6pt] &\quad\quad -1 \times 2 \times 0 - 1 \times 2 \times 0 - 5 \times 0 \times 4 \\[6pt] &=8. \end{aligned}\]
Problem 8.11
Evaluate the determinant of the matrix
\[A= \left( \begin{array}{crcc}
2 & -1 & 3 & 1 \\
0 & 2 & 2 & 0 \\
3 & 0 & 1 & 0 \\
0 & 0 & 2 & 0
\end{array}\right).\]
Solution. \[\begin{aligned} \det(A) &= -1 \times \det(\partial_{14} A) \\[6pt] &= -1 \times \det\left[\begin{array}{ccc} 0 & 2 & 2 \\ 3 & 0 & 1 \\ 0 & 0 & 2 \end{array}\right]\\[6pt] &= -1 \times 2 \times \det\left[\begin{array}{cc} 0 & 2 \\ 3 & 0 \end{array}\right] \\[6pt] &= -1 \times 2 \times (-6) = 12. \end{aligned}\]
Problem 8.12
Evaluate the determinant of the matrix
\[A= \begin{pmatrix}
2 & 6 & 1 & 8 & 0 \\
0 & 4 & 5 & 7 & 1 \\
0 & 0 & 4 & 9 & 7 \\
0 & 0 & 0 & 1 & 5 \\
0 & 0 & 0 & 1 & 0
\end{pmatrix}.\]
Solution. \[\begin{aligned} \det(A) &=-\det \left[\begin{array}{ccccc} 2 & 6 & 1 & 0 & 8 \\ 0 & 4 & 5 & 1 & 7 \\ 0 & 0 & 4 & 7 & 9 \\ 0 & 0 & 0 & 5 & 1 \\ 0 & 0 & 0 & 0 & 1 \end{array}\right] \\[6pt] &= -2 \times 4 \times 4 \times 5 \times 1 = -160. \end{aligned}\]
Problem 8.13
Consider a matrix \(A = \left( a_{ij} \right) \in M_n (K)\) whose elements are all zero above the minor diagonal (from bottom left to top right). Give a succinct formula for the determinant of \(A.\)
Solution. We want to find the number of column-exchange operations to make \(A\) as an upper-triangular matrix. If \(n=1,\) then there is nothing to consider, hence assume that \(n\ge 2.\) If \(n=2,\) only a single operation \[A^1 \,\leftrightarrow\,A^2\] is needed. If \(n=3,\) then a single operation \[A^1 \,\leftrightarrow A^3\] is sufficient. If \(n=4\) or \(n=5,\) then two operations \[A^1 \,\leftrightarrow \, A^n \quad\text{and}\quad A^2 \,\leftrightarrow\, A^{n-1}\] are needed.
In general, if \(n=4k\) or \(n=4k+1\) where \(k\) is a positive integer, then \(2k\) operations are needed to make \(A\) as an upper-triangular matrix, and if \(n=4k+2\) or \(n=4k+3,\) then \(2k+1\) operations are needed. Hence if \(n=4k\) or \(n=4k+1,\) then \[\det(A) = (\text{product of minor diagonal entries of } A),\] and if \(n=4k+2\) or \(n=4k+3,\) then \[\det(A) = -(\text{product of minor diagonal entries of } A).\]
Problem 8.14
Let \(A\in M_n (K)\) have factorization \(A=LU\) into the product of a lower triangular matrix \(L\) and an upper triangular matrix \(U.\) Show that the determinant of \(A\) is equal to the product of the diagonal terms in both \(L\) and \(U.\)
Solution. \[\begin{aligned} \det(A) &= \det(LU)\\[6pt] &= \det(L)\cdot\det(U) \\[6pt] &= (\text{product of diagonal entries of }L) \times \\[6pt] &\quad\quad(\text{product of diagonal entries of }U). \end{aligned}\]
Problem 8.15
Let \(A,\,B\in M_n(K)\) and suppose that \(A\) is singular, i.e., not invertible. Show that the product \(AB\) is also singular.
Solution. Since \(\det(A) =0,\) we have \[\det(AB) = \det(A)\det(B) = 0\times \det(B) =0.\]
Problem 8.16
Suppose that \(A\) is an invertible matrix such that both \(A\) and \(A^{-1}\) consist entirely of integers. Show that the determinant of \(A\) is either \(+1\) or \(-1.\)
Solution. Let \(\det(A) =p,\) \(\det(A^{-1})=q.\) Since \(A\) and \(A^{-1}\) are integer matrices, both \(p\) and \(q\) are integers. Besides, since \[pq = \det(A)\det(A^{-1}) = \det(AA^{-1}) = \det(I_n) = 1,\] we have \(p=\pm 1.\)
Problem 8.17
Consider each of the elementary row operations(defined on matrices in Section 5.3). Analyze how each affects the determinant of a square matrix.
Solution. (i) Interchanging two rows changes the sign of the determinant.
(ii) Multiplication of a row by a scalar \(k\) multiplies the determinant by \(k.\)
(iii) Addition of a scalar multiple of one row to another changes nothing of the determinant. (See Problem 5.)
Problem 8.18
For all \(A\in M_n(\mathbb{R}),\) prove that \(\det (AA^T ) \ge 0.\)
Solution. \[\begin{aligned} \det(AA^T) &= \det(A) \det(A^T) \\[6pt] &= \det(A) \det(A) \\[6pt] &= (\det(A))^2 \ge 0. \end{aligned}\]
Problem 8.19
Prove that if \(A\) is similar to \(B,\) then \(\det (A) = \det (B).\)
Solution. By the definition of similarity, there is an invertible matrix \(P\) such that \[A = P^{-1}BP.\] Hence \[\begin{aligned} \det(A) &= \det(P^{-1}BP) \\[6pt] &= \det(P^{-1})\det(B)\det(P) \\[6pt] &= (\det(P))^{-1} \det(B) \det(P) \\[6pt] &= \det(B). \end{aligned}\]
Problem 8.20
Prove that the following matrices are not similar.
\[A = \left( \begin{array}{crc} 1 & -1 & 0 \\ 0 & 2 & 5 \\ 0 & 0 & 3 \end{array}\right) ,\quad
B = \left( \begin{array}{rcc} 2 & 0 & 0 \\ -1 & 4 & 0 \\ 0 & 3 & 7 \end{array}\right).\]
Solution. Since \[\det(A) = 6 \ne 56 = \det(B),\] two matrices are not similar. (By the preceding problem.)
Problem 8.21
Determine whether the following set of vectors is linearly independent.
\[(1,\,2,\,-1),\,\, (6,\,0,\,2),\,\, (4,\,-4,\,2).\]
Solution. Let \[A=\left[\begin{array}{rcr} 1 & 6 & 4 \\ 2 & 0 & -4 \\ -1 & 2 & 2 \end{array}\right].\] Then \(\det(A) = 24 \ne 0.\) Hence the family of column vectors of \(A\) is linearly independent, that is, the given vectors constitute a linearly independent family.
Problem 8.22
Show that
\[\det \begin{pmatrix} 1 & x_1 & x_1 ^2 \\ 1 & x_2 & x_2 ^2 \\ 1 & x_3 & x_3 ^2 \end{pmatrix}
= \left( x_2 - x_1 \right) \left( x_3 - x_1 \right) \left( x_3 - x_2 \right).\]
Conclude that the vectors
\[ (1,\,x_1 ,\,x_1^2 ),\,\, (1,\,x_2 ,\,x_2^2 ) ,\,\, (1 ,\,x_3 ,\,x_3 ^2 )\]
are linearly independent if and only if \(x_1 ,\) \(x_2 \) and \(x_3\) are distinct. Generalize this to higher dimensions.
Solution. We immediately give the formula for the general case. Let \[A = \left[\begin{array}{ccccc} 1 & x_1 & x_1^2 & \cdots & x_1^{n-1} \\ 1 & x_2 & x_2^2 & \cdots & x_2^{n-1} \\ 1 & x_3 & x_3^2 & \cdots & x_3^{n-1} \\ \vdots & \vdots & \vdots & \ddots & \vdots \\ 1 & x_n & x_n^2 & \cdots & x_n^{n-1} \end{array}\right].\] As a polynomial of \(x_1,\) \(x_2,\) \(\cdots,\) \(x_n,\) the degree of \(\det(A)\) is \[1+2+3+\cdots + (n-1) = \frac{n(n-1)}{2}.\] IF \(x_i = x_j\) for \(i > j,\) then \(det(A) =0;\) Hence \(\det(A)\) has the factor \((x_i - x_j )\) for all \(i,\) \(j\) such that \(n \ge i > j \ge 1.\) Consider \[p(x_1 ,\,x_2 ,\, \cdots ,\,x_n ) = \prod_{i > j}(x_i - x_j).\] Then the degree of \(p\) is \(n(n-1)/2\) which coincides the degree of \(\det(A),\) and \(p\) has the factor \((x_i - x_j )\) for all \(i,\) \(j\) such that \(n \ge i > j \ge 1.\) Hence \[\det(A) = kp(x_1 ,\,x_2 ,\, \cdots ,\,x_n )\] for some \(k.\)
If we take the first term of each factor of \(p,\) then their product becomes \[x_2 \, x_3 ^2 \, x_4^3 \cdots x_n^{n-1}\] which is the product of diagonal entries of \(A.\) This term is unique among the expansion of \(p,\) so is in \(\det(A).\) Hence \(k=1.\)
We conclude that the family of column vectors \(A^1 ,\) \(A^2 ,\) \(\cdots ,\) \(A^n\) is linearly independent if and only if all \(x_j ' \text{s}\) are distinct.
Problem 8.23
Let \(f_1,\) \(\cdots ,\) \(f_n\) be family of functions in \(C^{\infty} (\mathbb{R}).\) Note that if
\[\sum_{j=1}^n \lambda_j f_j (x) =0\]
for some family of coefficients \(\lambda_j \in \mathbb{R},\) then
\[\sum_{j=1}^n \lambda_j f_j ^{(k)} (x) =0\]
for all \(k\ge 0.\) With this background, show that if \(f_1,\) \(\cdots,\) \(f_n\) are linearly dependent, then
\[\det \begin{pmatrix}
f_1 & f_2 & \cdots & f_n \\
f_1^{(1)} & f_2^{(1)} & \cdots & f_n^{(1)} \\
\vdots & \vdots & \ddots & \vdots \\
f_1^{(n-1)} & f_2^{(n-1)} & \cdots & f_n^{(n-1)}
\end{pmatrix} =0.\]
(This is called the Wronskian determinant of the family \(f_1,\) \(\cdots,\) \(f_n\) and is critical in the theory of differential equation.)
Solution. Since the family \(f_1 ,\) \(f_2 ,\) \(\cdots ,\) \(f_n\) is linearly dependent, there exist scalars \(\lambda_j ' \text{s}\) such that \[\lambda_1 f_1 + \lambda_2 f_2 + \cdots + \lambda_n f_n = 0\] and not all \(\lambda_j ' \text{s}\) are zeros. Let \[A=\left[\begin{array}{ccccc} f_1 & f_2 & f_3 & \cdots & f_n \\ f_1 ^{(1)} & f_2 ^{(1)} & f_3 ^{(1)} & \cdots & f_n ^{(1)} \\ f_1 ^{(2)} & f_2 ^{(2)} & f_3 ^{(2)} & \cdots & f_n ^{(2)} \\ \vdots & \vdots & \vdots & \ddots & \vdots \\ f_1 ^{(n-1)} & f_2 ^{(n-1)} & f_3 ^{(n-1)} & \cdots & f_n ^{(n-1)} \end{array}\right].\] Let \(A^j\) be the \(j\)th column of \(A.\) Then \[\lambda_1 A^1 + \lambda_2 A^2 + \cdots + \lambda_n A^n = 0.\] Hence the family of column vectors of \(A\) is linearly dependent. Consequently we have \(\det(A) =0.\)
Problem 8.24
Use the previous exercise to show that the functions \(e^x,\) \(e^{2x}\) and \(e^{3x}\) are linearly independent; that is, show that
\[\det \begin{pmatrix} e^x & e^{2x} & e^{3x} \\ e^x & 2e^{2x} & 3e^{3x} \\ e^x & 4e^{2x} & 9e^{3x} \end{pmatrix} \ne 0.\]
Solution. \[\begin{aligned} \det \left[\begin{array}{ccc} e^x & e^{2x} & e^{3x} \\ e^x & 2e^{2x} & 3e^{3x} \\ e^x & 4e^{2x} & 9e^{3x} \end{array}\right] &= \det \left( e^x e^{2x} e^{3x} \left[\begin{array}{ccc} 1 & 1 & 1 \\ 1 & 2 & 3 \\ 1 & 2^2 & 3^2 \end{array}\right] \right) \\[6pt] &= e^x e^{2x} e^{3x} \det \left[ \begin{array}{ccc} 1 & 1 & 1 \\ 1 & 2 & 3 \\ 1 & 2^2 & 3^2 \end{array}\right] \ne 0. \end{aligned}\]
Problem 8.25
Use Problem 22 and 23 to show that the collection of functions
\[e^{\lambda_1 x}, \,\, e^{\lambda_2 x} ,\,\, \cdots ,\,\, e^{\lambda_n x} \,\,\, ( \lambda_j \in \mathbb{R} )\]
is linearly independent if and only if the numbers \(\lambda_j\) are distinct. This generalizes the previous problem.
Solution. If \(\lambda_i = \lambda_j\) for some \(i\ne j,\) then the family of given functions is linearly dependent.
Suppose now that \(\lambda_j ' \text{s}\) are all distinct. Let \[A = \left[\begin{array}{ccccc} e^{\lambda_1 x} & e^{\lambda_2 x} & e^{\lambda_3 x} & \cdots & e^{\lambda_n x} \\ \lambda_1 e^{\lambda_1 x} & \lambda_2 e^{\lambda_2 x} & \lambda_3 e^{\lambda_3 x} & \cdots & \lambda_n e^{\lambda_n x} \\ \lambda_1 ^2 e^{\lambda_1 x} & \lambda_2 ^2 e^{\lambda_2 x} & \lambda_3 ^2 e^{\lambda_3 x} & \cdots & \lambda_n ^2 e^{\lambda_n x} \\ \vdots & \vdots & \vdots & \ddots & \vdots \\ \lambda_1 ^{n-1} e^{\lambda_1 x} & \lambda_2 ^{n-1} e^{\lambda_2 x} & \lambda_3 ^{n-1} e^{\lambda_3 x} & \cdots & \lambda_n ^{n-1} e^{\lambda_n x} \\ \end{array}\right].\] Then \[\det(A) = e^{\lambda_1 x} e^{\lambda_2 x} \cdots e^{\lambda_n x} \det\left[\begin{array}{ccccc} 1 & 1 & 1 & \cdots & 1 \\ \lambda_1 & \lambda_2 & \lambda_3 & \cdots & \lambda_n \\ \lambda_1 ^2 & \lambda_2 ^2 & \lambda_3 ^2 & \cdots & \lambda_n ^2 \\ \vdots & \vdots & \vdots & \ddots & \vdots \\ \lambda_1 ^{n-1} & \lambda_2 ^{n-1} & \lambda_3 ^{n-1} & \cdots & \lambda_n ^{n-1} \\ \end{array}\right] \ne 0\] by the Problem 22. Hence by Problem 23, the family of \(e^{\lambda_j x}\)'s is linearly independent.
Problem 8.26
Let
\[A= \begin{pmatrix} 1 & 0 & 1 \\ 0 & 2 & 1 \\ 1 & 4 & 3 \end{pmatrix}.\]
Find \(A^{-1}\) or prove that it does not exist.
Solution. If \(A^j\) is the \(j\)th column vector of \(A,\) then \[2A^1 + A^2 = 2A^3,\] that is, the family of column vectors of \(A\) is not linearly independent. Hence \(\det(A) =0\) and \(A^{-1}\) does not exist.
Problem 8.27
Show that \(\operatorname{SL}_n(K)\) is a subgroup of \(\operatorname{GL}_n(K).\)
Solution. If \(A,\) \(B\in \operatorname{SL}_n (K),\) then \[\det(AB) = \det(A)\det(B) = 1\times 1 = 1,\] hence \(AB\in\operatorname{SL}_n (K).\)
If \(A\in\operatorname{SL}_n (K),\) then \(\det(A) =1.\) Hence \(A^{-1}\) exists and \[\det(A^{-1}) = (\det(A))^{-1} = 1^{-1} = 1,\] that is, \(A^{-1} \in \operatorname{SL}_n (K).\)
Therefore \(\operatorname{SL}_n(K)\) is a subgroup of \(\operatorname{GL}_n(K).\)
Problem 8.28
Consider \(2\times 2\) matrix
\[A = \begin{pmatrix} a & b \\ c & d \end{pmatrix}.\]
Let \(P = (a,\,c)\) and \(Q = (b,\,d)\) be the points in \(\mathbb{R}^2\) defined by the columns of \(A.\) These points span a parallelogram whose vertices are \(\mathbf{0},\) \(P,\) \(Q\) and \(P+Q.\) Find the area of this object.
Solution. Keep calm and sketch a picture to derive \[\left\lvert \det(A) \right\rvert.\]